看板 Physics 關於我們 聯絡資訊
推 yvb: 從數字板得知原PO的小說是極限返航. 03/31 19:26 → yvb: https://www.books.com.tw/products/0010916490 03/31 19:26 → yvb: 預覽第三頁可能就是太空船和變形圖. https://reurl.cc/Go8j53 03/31 19:29 → yvb: 不知小說的原文如何寫, 而該圖也沒尺寸比例, 不知從何算起. 03/31 19:39 → yvb: 至少我不認為配重方式沒變. 03/31 19:40 有這圖的話可以簡單概算一下 展開前視為均質,展開後中間幾乎無質量 前後平均各長20公尺,再對半(坐在正中間)為10公尺,對上75公尺可忽視 用的公式是角動量守恆L=Iω L = Iω= (0.5M*R1^2)*ω1 = (M*R2^2)*ω2,a1 = R1*ω1,a2 = R2*ω2 換算完並代入數字可以得到 a2/a1 = 0.25*(20/75)^3 然後會發現其實0.25比20/75還小,不過剛剛有忽視那十公尺 所以可以說答案估計會接近(20/75)的四次方吧 如果他配重的改變方式是反過來的話就真的會接近平方了XD 不忽視用20/65下去算,加速度比約為0.00728 (20/75)^4約為0.00506,以這種估算法來說我認為這樣的差距可接受 我不想用空心有厚度圓柱體算,懶XD -- https://i.imgur.com/h4Q0F04.jpg
9月23日 發生大事了 因為就在這天,加藤惠誕生了 https://i.imgur.com/H3RhXfJ.jpg
-- ※ 發信站: 批踢踢實業坊(ptt.cc), 來自: 223.138.55.43 (臺灣) ※ 文章網址: https://www.ptt.cc/bbs/Physics/M.1648789540.A.E80.html ※ 編輯: fragmentwing (223.138.55.43 臺灣), 04/01/2022 13:14:10
selfvalue: 推 04/02 01:31
yvb: 原PO可能要先搞清楚原原PO的問題,和太空船的圖. 04/06 23:13
yvb: 不然只是一廂情願地算出一個和原題無關的答案而已. 04/06 23:15
yvb: 1. 駕駛艙離旋轉中心的半徑 ... 為何還要對半? 04/06 23:17
yvb: 2. 看一下太空船的圖, 駕駛艙的位置在何處... 04/06 23:19
yvb: 3. 不知剩餘燃料量, 也不知其機構, 質心在何處? 可視為均質? 04/06 23:21
fragmentwing: 因為駕駛人不會在75公尺的端點上 所以估實際距離會 04/07 11:13
fragmentwing: 在對半處 看圖的話可能會更接近圓心一點 但我沒估到 04/07 11:13
fragmentwing: 那麼細 04/07 11:13
fragmentwing: 質心部份你會看到我是把拆開後對質量合計為M,距軸 04/07 11:15
fragmentwing: 心R2的兩顆質心進行計算 04/07 11:15
fragmentwing: 因為對比的估算是一句平方比而已 你要考量那麼多反 04/07 11:17
fragmentwing: 而是搞錯題旨 04/07 11:17
yvb: 首先, 除非駕駛人不在駕駛艙, 否則就是 20 和 75. 04/08 23:05
yvb: 再來, I 的估算誤差很大.請到wiki查詢轉動慣量列表就知. 04/08 23:07
yvb: 即使是不同次方, 由於數值差異不是很大, 會因為乘上的係數差, 04/08 23:14
yvb: 而得到截然不同的數值. 04/08 23:16
yvb: 你的計算, 就如同我在三樓所說的那樣. 04/08 23:17
fragmentwing: 你用75代表你人會跑到火箭頂部開船 65代表的是更進 04/09 15:38
fragmentwing: 一步的細算了 而原po提及原作的那種估算 法 實際上 04/09 15:38
fragmentwing: 要是三次方*0.25才對 04/09 15:38
fragmentwing: 你真的搞錯題旨了 04/09 15:38
yvb: 請把圖看清楚,駕駛艙不在頂端,而在三角錐的底部,減壓艙另一側 04/11 21:12
yvb: 基本上是三次方,但0.25??? 此外不能視為只有駕駛艙的質點. 04/11 21:15
yvb: 若另有一人在儲藏艙處,設儲藏艙距駕駛艙20M,用一樣的算法, 04/11 21:18
yvb: 會算出什麼結果? 再回推兩處的角速度變化, 看是否會有問題? 04/11 21:19
yvb: 也就是說, 這個三次方應以何處做為基準來計算, 就是個問題了. 04/11 21:21
yvb: 是否均質,算出來的轉動慣量也會有很大差別. 04/11 21:27
fragmentwing: 我說了 你的算法也許更正確 但考慮這些會違背題目的 04/14 18:21
fragmentwing: 初衷 04/14 18:21